Đến nội dung

chardhdmovies nội dung

Có 621 mục bởi chardhdmovies (Tìm giới hạn từ 29-04-2020)



Sắp theo                Sắp xếp  

#531345 Yên Bái TST 2015

Đã gửi bởi chardhdmovies on 01-11-2014 - 05:20 trong Thi HSG cấp Tỉnh, Thành phố. Olympic 30-4. Đề thi và kiểm tra đội tuyển các cấp.

Đề này ảo thật, bài 3 là bổ đề đường thẳng Simson , bài 4 số học lớp 9 dùng tính chia hết , bài 5 là đề thi hsg toán tp hà nội năm 2013 khi thay 2015 bởi 2013 . Bài số 1 ngày 2 thi đã giải nhiều lần trên diễn đàn ,bài 3 thì nằm trong nâng cao pt toán lớp 9 tập 2 

bài nào vậy ạ

 

NTP




#523790 với $x,y,z\geq 0$

Đã gửi bởi chardhdmovies on 10-09-2014 - 19:12 trong Bất đẳng thức - Cực trị

với $x,y,z\geq 0$

$$CM \left ( xy+yz+zx \right ) \left ( \frac{1}{\left ( x-y \right )^{2}} +\frac{1}{\left ( y-z \right )^{2}} +\frac{1}{\left ( z-x \right )^{2}} \right)$$$\geq 4$

không mất tính tổng quát gải sử $z=min\left \{ x,y,z \right \}$

ta có $\sum \frac{1}{(x-y)^2}=\frac{1}{(x-y)^2}+\frac{(x-y)^2}{(y-z)^2(z-x)^2}+\frac{2}{(z-x)(z-y)}\geq \frac{4}{(z-x)(z-y)}$

ta chứng minh $(z-x)(z-y)\leq xy+yz+zx\Leftrightarrow z(2x+2y-z)\geq 0$

điều này luôn đúng nên có đpcm

dấu bằng xảy ra khi $x=\frac{3\pm \sqrt{5}}{2}y;z=0$ và các hoán vị

 

NTP




#526149 VỀ ỨNG DỤNG CỦA MỘT BẤT ĐẲNG THỨC NHỎ

Đã gửi bởi chardhdmovies on 25-09-2014 - 20:37 trong Chuyên đề toán THPT

Một bài toán có sử dụng BĐT $(*)$ là:

Cho các số thực dương $a, b, c$ thỏa mãn 

$$3\left( {{a^2} + {b^2} + {c^2}} \right) + ab + bc + ca = 12$$

Chứng minh rằng 

$${a \over {\sqrt {a + b} }} + {b \over {\sqrt {b + c} }} + {c \over {\sqrt {c + a} }} \le {3 \over {\sqrt 2 }}$$

ta có $(\sum \frac{a}{\sqrt{a+b}})^2\leq [\sum a(a+c)][\sum \frac{a}{(a+b)(a+c)}]=\frac{2(\sum a^2+\sum ab)(\sum ab)}{\prod (a+b)}$

                                                       $\leq \frac{9}{4}\frac{( a^2+b^2+c^2+ab+bc+ca)}{a+b+c}$

ta chứng minh $\frac{a^2+b^2+c^2+ab+bc+ca}{a+b+c}\leq 2$

mà $ a^2+b^2+c^2+ ab+bc+ca=\frac{12+2(a+b+c)^2}{5}$

do đó ta sẽ chứng minh $\frac{12+2(a+b+c)^2}{5(a+b+c)}\leq 2\Leftrightarrow 2\leq a+b+c\leq 3$            $(*)$

mà $3(a+b+c)^2-12=3(\sum a)^2-(3\sum a^2+\sum ab)=5(ab+bc+ca)>0\Rightarrow a+b+c>2$

$4(\sum a)^2-36=4(\sum a)^2-3(3\sum a^2+\sum ab)=-5(\sum a^2-\sum ab)\leq 0\Rightarrow a+b+c\leq 3$

do đó $(*)$ được chứng minh nên bđt được chứng minh

 

NTP




#525930 VỀ ỨNG DỤNG CỦA MỘT BẤT ĐẲNG THỨC NHỎ

Đã gửi bởi chardhdmovies on 24-09-2014 - 04:56 trong Chuyên đề toán THPT

 

VỀ ỨNG DỤNG CỦA MỘT BẤT ĐẲNG THỨC NHỎ

 

      
$\boxed{1}$: Cho $a, b, c$ là các số thực dương. Chứng minh rằng
$${1 \over {2a + b}} + {1 \over {2b + c}} + {1 \over {2c + a}} \ge {2 \over {\root 3 \of {\left( {a + b} \right)\left( {b + c} \right)\left( {c + a} \right)} }}$$
 
$\boxed{2}$: Cho $a, b, c$ là các số thực dương thỏa mãn $abc=1$. Chứng minh rằng
$$4\left( {\root 3 \of {{a \over b}}  + \root 3 \of {{b \over c}}  + \root 3 \of {{c \over a}} } \right) \le 3\root 3 \of {{{\left( {2 + a + b + c + {1 \over a} + {1 \over b} + {1 \over c}} \right)}^2}} $$
 
---------Hết---------

 

$1$

nhân cả hai vế bđt với $2(a+b+c)$ thì ta cần chứng minh $\sum_{cyc}^{.}\frac{b}{b+2a}+2\sum_{cyc}^{.}\frac{a}{2b+c}+3\geq \frac{4(a+b+c)}{\sqrt[3]{(a+b)(b+c)(c+a)}}$

ta có $\sum_{cyc}^{.}\frac{b}{b+2a}\geq 1,\sum_{cyc}^{.}\frac{a}{2b+c}\geq \frac{(a+b+c)^2}{3(ab+bc+ca)}$

do đó ta cần chứng minh $\frac{(a+b+c)^2}{3(ab+bc+ca)}+2\geq \frac{2(a+b+c)}{\sqrt[3]{(a+b)(b+c)(c+a)}}$

mà ta có $\frac{(a+b+c)^2}{3(ab+bc+ca)}+1+\geq 3\sqrt[3]{\frac{(a+b+c)^2}{3(ab+bc+ca)}}$

do đó ta sẽ chứng minh $3\sqrt[3]{\frac{(a+b+c)^2}{3(ab+bc+ca)}}\geq \frac{2(a+b+c)}{\sqrt[3]{(a+b)(b+c)(c+a)}}\Leftrightarrow 9\prod (a+b)\geq 8(\sum ab)(\sum a)$

điều này luôn đúng nên có đpcm

$2$

đặt $a=\frac{y}{x},b=\frac{z}{y},c=\frac{x}{z}$ do đó cần chứng minh $\frac{4(x+y+z)}{\sqrt[3]{xyz}}\leq 3[\frac{(x+y)(y+z)(z+x)}{xyz}]^{\frac{2}{3}}$

$\Leftrightarrow xyz(x+y+z)^3\leq \frac{27}{64}(x+y)^2(y+z)^2(z+x)^2$

mà $\frac{27}{64}\prod (x+y)^2\leq \frac{1}{3}(\sum x)^2(\sum xy)^2\geq xyz(x+y+z)^3$

do đó có được điều cần chứng minh

 

 

NTP




#525970 Tuyển tập một số bài phương trình, hệ phương trình thi HSG tỉnh

Đã gửi bởi chardhdmovies on 24-09-2014 - 16:54 trong Phương trình - hệ phương trình - bất phương trình

Giải hệ PT

 

1. $\left\{ \begin{matrix} 6{x^2} - y - x{y^2} = 0\\ 5{x^2} - {x^2}{y^2} - 1 = 0 \end{matrix} \right.$

 

2. $\left\{ \begin{matrix} 2{x^3} - 9{y^3} = \left( {x - y} \right)\left( {2xy + 3} \right)\\ {x^2} - xy + {y^2} = 3 \end{matrix} \right.$

 

3. $\left\{ \begin{matrix}\left( {x + \sqrt {{x^2} + 1} } \right)\left( {y + \sqrt {{y^2} + 1} } \right) = 1\\ \sqrt {2x - 1} + 1 = y\left( {x - 3} \right) \end{matrix} \right.$

$1$

hpt tương đương $\left\{\begin{matrix} xy^2+y=6x^2\\x^2y^2+1 =5x^2 \end{matrix}\right.\Leftrightarrow \left\{\begin{matrix} \frac{y}{x}(\frac{1}{x}+y)=6\\(\frac{1}{x}+y)^2-2\frac{y}{x}=5 \end{matrix}\right.$

phần còn lại đặt ẩn là được

$2$

thay $3$ từ $PT(2)$ vào $PT(1)$ ta được $2x^3-9y^3=(x-y)(x^2+xy+y^2)\Leftrightarrow x^3=8y^3\Leftrightarrow x=2y$

tới đây thế vào $PT(2)$ là được

$3$

từ $PT(1)$ suy ra $x+y=0$ nên ta có $PT(2)$ là $(x-\frac{1}{2})^2=(\sqrt{2x-1}-\frac{1}{2})^2$

phần còn lại ok rồi

 

NTP




#517498 Trại Hè Phương Nam $2014$ - OlymPic Toán Học Tỉnh Đồng Tháp

Đã gửi bởi chardhdmovies on 03-08-2014 - 22:04 trong Thi HSG cấp Tỉnh, Thành phố. Olympic 30-4. Đề thi và kiểm tra đội tuyển các cấp.

2,

ta có $(-x)^2-x=3-2x+\sqrt{3-2x}$

tới đây đặt ẩn hoặc xét hàm $f(t)=t^2+t$ là được




#517497 Trại Hè Phương Nam $2014$ - OlymPic Toán Học Tỉnh Đồng Tháp

Đã gửi bởi chardhdmovies on 03-08-2014 - 22:03 trong Thi HSG cấp Tỉnh, Thành phố. Olympic 30-4. Đề thi và kiểm tra đội tuyển các cấp.

3,Capture.PNG

 




#524285 TOPIC: Các bài toán có nội dung hình học phẳng tuyển chọn

Đã gửi bởi chardhdmovies on 13-09-2014 - 20:26 trong Hình học

Bài 3 Chứng minh rằng nếu một đường thẳng đồng thời chia chu vi và diện tích tam giác thành 2 phần bằng nhau thì đường thẳng đó đi qua tâm đường tròn nội tiếp tam giác

Bài 4: ( Đề thi HSG cấp tỉnh Thanh Hóa 2006) Cho tam giác ABC. Đường trung tuyến AD,đường cao BH và đường phân giác CE đồng quy. Chứng minh hệ thức $(a+b)(a^{2}+b^{2}-c^{2})=2a^{2}b$

$3,$có thể mở rộng với tứ giác

$4,$ 

xem lại đề đi hình như là đường cao $AD$,trung tuyến $BH$ và phân giác $CE$

 

NTP




#524506 TOPIC: Các bài toán có nội dung hình học phẳng tuyển chọn

Đã gửi bởi chardhdmovies on 14-09-2014 - 18:39 trong Hình học

Bài 9: Cho tam giác $ABC$ có $\widehat{A}=90^{\circ}$. Hình vuông $MNPQ$ nội tiếp tam giác ABC sao cho $M$ nằm trên đoạn $AB$, $N$ nằm trên đoạn $AC$ và $P,Q$ nằm trên đoạn $BC$. Chứng minh rằng $BC \geq 3QP$. Dấu $=$ xảy ra khi nào?

Capture.PNG

gọi $D$ là trung điểm $BC$ ta có $AH\leq AD=\frac{1}{2}BC$

gọi $H$ là hình chiếu của $A$ trên $BC$

ta có $\frac{BM}{AB}=\frac{MQ}{AH};\frac{AM}{AB}=\frac{MN}{BC}$

$\Rightarrow 1=(\frac{QM}{2AH}+\frac{QM}{2AH}+\frac{MN}{BC})^3\geq \frac{27QM^2MN}{4AH^2.BC}\geq \frac{27PQ^3}{4.(\frac{BC}{2})^2.BC}\Rightarrow BC\geq 3PQ$

dấu bằng xảy ra khi $\Delta ABC$ vuông cân

 

NTP




#524278 TOPIC: Các bài toán có nội dung hình học phẳng tuyển chọn

Đã gửi bởi chardhdmovies on 13-09-2014 - 20:05 trong Hình học

Bài 2: Cho tam giác ABC không cân,đường trung tuyến AD, đường phân giác AE. Từ C kẻ đường thẳng vuông góc với AE cắt AE,AD,AB tại F,G,K. Chứng minh rằng DF đi qua trung điểm GE

Capture.PNG

gọi $FD\cap AC\in \left \{ H \right \}$

dễ thấy $\Delta AKC$ cân tại $A$ và $FD$ là đường trung bình $\Delta BCK$ nên $\widehat{AFH}=\widehat{KAF}=\widehat{FAC}\Rightarrow \left\{\begin{matrix} AH=HF\\\widehat{HFC}=\widehat{HCF} \end{matrix}\right.$ do đó $H$ là trung điểm $AC$

áp dụng đinh lí $ceva$ trong $\Delta AFC\Rightarrow \frac{EA}{EF}.\frac{GF}{GC}.\frac{HC}{HA}=1\Rightarrow \frac{EA}{EF}=\frac{GC}{GF}\Rightarrow EG//AC$

do đó dễ thấy $DF$ đi qua trung điểm $GE$

 

NTP




#524324 TOPIC: Các bài toán có nội dung hình học phẳng tuyển chọn

Đã gửi bởi chardhdmovies on 13-09-2014 - 22:08 trong Hình học

Bài 1 đâu cần phải tam giác nhọn đâu nhỉ?

 

nhầm

Bài 4: ( Đề thi HSG cấp tỉnh Thanh Hóa 2006) Cho tam giác ABC. Đường trung tuyến AD,đường cao BH và đường phân giác CE đồng quy. Chứng minh hệ thức $(a+b)(a^{2}+b^{2}-c^{2})=2ab^{2}$

Capture.PNG

gọi giao điểm $AD,BH,CE$ là $O$

gọi $I$ là hình chiếu từ $D$ xuống $BH$ do đó $DI=\frac{1}{2}CH$

có $\frac{DI}{AH}=\frac{DO}{AO}=\frac{DC}{AC}\Rightarrow \frac{HC}{AH}=\frac{a}{b}\Rightarrow bHC=aAH$

có $\left\{\begin{matrix} a^2=BH^2+HC^2\\b^2=AH^2+HC^2+2AH.HC \\c^2=AH^2+BH^2 \end{matrix}\right.$

$\Rightarrow (a+b)(a^2+b^2-c^2)=(a+b)(2bHC)=2b(aHC+aAH)=2ab^2$

 

NTP




#524552 TOPIC: Các bài toán có nội dung hình học phẳng tuyển chọn

Đã gửi bởi chardhdmovies on 14-09-2014 - 21:26 trong Hình học

Bài 11: (Kết bài này) Cho lục giác lồi $ABCDEF$ có $AB=BC,CD=DE,EF=FA$. Chứng minh rằng: $\frac{BC}{BE}+\frac{DE}{DA}+\frac{FA}{FC}\geq \frac{3}{2}$. Dấu bằng xảy ra khi nào?

Capture.PNG

đặt $AC=x;CE=y;EA=z$

theo bđt $ptoleme$ cho tứ giác $ACEF$ ta có  $AC.EF+CE.AF\geq AE.CF\Rightarrow FA(x+y)\geq FCz\Rightarrow \frac{FA}{FC}\geq \frac{z}{x+y}$

tương tự thì ta có $\frac{DE}{DA}\geq \frac{y}{z+x};\frac{BC}{BE}\geq \frac{x}{y+z}$

do đó $\frac{BC}{BE}+\frac{DE}{DA}+\frac{FA}{FC}\geq \frac{x}{y+z}+\frac{y}{z+x}+\frac{z}{x+y}\geq \frac{3}{2}$

dấu bằng xảy ra khi $ABCDEF$ là lục giác đều

 

bđt chặt hơn cho bài trên là $\frac{BC}{BE}+\frac{DE}{DA}+\frac{FA}{FC}\geq \frac{3}{2}+\frac{(AC-CE)^2+(CE-AE)^2+(AE-AC)^2}{(AC+CE)^2+(CE+AE)^2+(AE+AC)^2}$

 

NTP




#524987 TOPIC: Các bài toán có nội dung hình học phẳng tuyển chọn

Đã gửi bởi chardhdmovies on 17-09-2014 - 18:21 trong Hình học

Bài 24: (Đề thi toán quốc tế IMO 2013,câu 3) . Cho tam giác $ABC$. Gọi $A_1,B_1,C_1$ thứ tự là các tiếp điểm của đường tròn bàng tiếp tam giác với các cạnh $BC,CA,AB$. Chứng minh rằng nếu tâm đường tròn ngoại tiếp tam giác $A_1B_1C_1$ nằm trên đường tròn ngoại tiếp tam giác $ABC$ thì tam giác $ABC$ là tam giác vuông.

xem ở đây 

 

 

NTP




#524557 TOPIC: Các bài toán có nội dung hình học phẳng tuyển chọn

Đã gửi bởi chardhdmovies on 14-09-2014 - 21:47 trong Hình học

bài 13 : cho $\Delta ABC$ có $\widehat{BAC}=105^0$,đường trung tuyến $BM$ và đường phân giác $CD$ cắt nhau tại $K$ sao cho $KB=KC$.Gọi H là hình chiếu của $A$ xuống $BC$.

CMR $HA=HB$

bài 14 :cho tứ giác $ABCD$ nội tiếp đường tròn $(O)$.Một đường thẳng qua $C$ cắt các tia đối của tia $AB,AD$ lần lượt tại $M,N$

CMR: $\frac{4S_{BCD}}{S_{AMN}}\leq (\frac{BD}{CD})^2$

 

NTP




#524565 TOPIC: Các bài toán có nội dung hình học phẳng tuyển chọn

Đã gửi bởi chardhdmovies on 14-09-2014 - 22:13 trong Hình học

$\blacksquare$ bổ đề $1$:trong $\Delta ABC$ $\widehat{BAC}\geq 90^0\Leftrightarrow BC^2\geq AB^2+AC^2$

$\blacksquare$  bổ đề $2$:trong tứ giác $ABCD$ với $I;J$ là trung điểm $AC,BD$ thì $AB^2+BC^2+CD^2+DA^2=AC^2+BD^2+4IJ^2$

Capture.PNG

$*$ quay lại bài toán:

từ bổ đề $2$  ta có $AC^2+BD^2\leq AB^2+BC^2+CD^2+DA^2$

$\Leftrightarrow \frac{\sqrt{AC^2+BD^2}}{2}\leq \frac{\sqrt{AB^2+BC^2+CD^2+DA^2}}{2}$

                                       $\leq\frac{ \sqrt{4max\left \{ AB^2;BC^2;CD^2;DA^2 \right \}}}{2}=max\left \{ AB,BC,CD,DA \right \}$

đặt $m=min\left \{ AB,BC,CD,DA \right \}$

không mất tính tổng quát $\widehat{BAD}+\widehat{ADC}\geq 180^0;\widehat{BAD}\geq 90^0$

dựng hình bình hành $ABED$ thì $DE$ nằm giữa $DB,DC$

gọi $I;J$ là trung điểm $AC,BD$

trong tam giác $ACE$ có $IJ$ là đường trung bình nên $CE=2IJ$

có hai trường hợp xảy ra

$\blacklozenge$ trường hợp $1$ :$E$ nằm trong tứ giác $ABCD$.Trong hai góc $\widehat{AEB};\widehat{AED}$ có ít nhất một góc nhọn

không mất tính tổng quát giả sử $\widehat{AEB}\leq 90^0\Rightarrow \widehat{CEB}\geq 90^0$

theo bổ đề $1$ thì $BC^2\geq BE^2+CE^2\Leftrightarrow BC^2-4IJ^2\geq AD^2$

theo bổ đề $2$ thì $AC^2+BD^2=AB^2+AD^2+CD^2+(BC^2-4IJ^2)\geq AB^2+AD^2+CD^2+AD^2\geq 4m^2$

$\Rightarrow m\leq \frac{\sqrt{AC^2+BD^2}}{2}$

$\blacklozenge$ trường hợp $2$:$E$ nằm ngoài tứ giác $ABCD$ 

khi đí $CB$ nằm giữa $CD,CE$.Do đó $\widehat{BEC}\geq \widehat{BED}=\widehat{BAD}\geq 90^0$

khi đó chứng minh tương tự trường hợp $1$

do đó bài toán được chứng minh

 

NTP




#525497 TOPIC: Các bài toán có nội dung hình học phẳng tuyển chọn

Đã gửi bởi chardhdmovies on 21-09-2014 - 12:01 trong Hình học

Bài 25: Cho đường tròn (O) và đường thẳng d nằm ngoài (O). Gọi D là hình chiếu của O trên d, M là một điểm di động trên d. Từ M kẻ 2 tiếp tuyến MA, MB với (O), F và E lần lượt là hình chiếu của D trên MA, MB. Chứng minh EF luôn đi qua 1 điểm cố định.

 

P/s: hãy nghĩ cách giải bài 22 đi mọi người 

Capture.PNG

gọi $EF\cap OD\in \left \{ I \right \};AB\cap OD\in \left \{ S \right \}$

một bài toán cơ bản là chứng minh $S$ cố định(cái này mọi người tự chứng minh nha,dùng phương tích)

gọi $G$ là hình chiếu của $D$ trên $AB$ thì đường thẳng $simon$ của $\Delta ABM$  là $EF$ do đó $E,F,G$ thẳng hàng

ta có $\widehat{IDG}=\widehat{MOD}=\widehat{MBD}=\widehat{IGD}$

do đó dễ dàng chứng minh $I$ là trung điểm $SD$ do đó $I$ cố định

 

NTP




#523835 Topic Đề thi THCS

Đã gửi bởi chardhdmovies on 10-09-2014 - 21:57 trong Tài liệu - Đề thi

10689733_1484590751808563_24866820663089

$3b$ ý sau

ta có $(1-a)(1-b)(1-c)\leq \frac{(3-a-b-c)^3}{27}=\frac{1}{27}$

$\Leftrightarrow \frac{-28}{57}+ab+bc+ca\leq abc\Leftrightarrow \frac{52}{27}\leq a^2+b^2+c^2+2abc$




#606966 TOPIC ôn thi Olimpic 30/04 và thi HSG toán 10

Đã gửi bởi chardhdmovies on 03-01-2016 - 15:46 trong Thi HSG cấp Tỉnh, Thành phố. Olympic 30-4. Đề thi và kiểm tra đội tuyển các cấp.

Nhiệt liệt ủng hộ topic này luôn, chúc topic phát triển và đóng góp nhiều bài tập hay  :lol:

Bài 2: Cho a, b, c, d là các số thực dương thỏa mãn đk: $\frac{1}{a} + \frac{1}{b} + \frac{1}{c} + \frac{1}{d} = 4$. CM:

$\sqrt[3]{\frac{a^{3} + b^{3}}{2}} + \sqrt[3]{\frac{b^{3} + c^{3}}{2}} + \sqrt[3]{\frac{c^{3} + d^{3}}{2}} + \sqrt[3]{\frac{d^{3} + a^{3}}{2}} \leq 2(a + b + c + d) - 4$

Bài 3: CMR nếu a,b,c>0 thì: 

$\frac{(a + b + c)^{2}}{ab + bc + ca} \geq \frac{a + b}{a + c} + \frac{b + c}{b + a} + \frac{c + a}{c + b}$

3. Giả sử $c=min\left \{ a,b,c \right \}$. Ta biến đổi BĐT như sau:

$\frac{(a+b+c)^{2}}{ab+bc+ca}-3\geq (\frac{a+b}{a+c}-1)+(\frac{b+c}{b+a}-1)+(\frac{c+a}{c+b}-1)$

$\Leftrightarrow \frac{a^2+b^2+c^2-ab-bc-ca}{ab+bc+ca}\geq \frac{b-c}{a+c}+\frac{c-a}{b+a}+\frac{a-b}{c+b}$

$\Leftrightarrow \frac{(a-b)^2+(a-c)(b-c)}{ab+bc+ca}\geq\frac{(a-c)(b-c)+b^2-a^2}{(a+c)(a+b)}+\frac{a-b}{b+c}$
$\Leftrightarrow \frac{(a-b)^2+(a-c)(b-c)}{ab+bc+ca}\geq\frac{(a-c)(b-c)}{(a+c)(a+b)}+\frac{(a-b)^2}{(a+c)(b+c)}$

$\Leftrightarrow \frac{c^{2}(a-b)^2}{(ab+bc+ca)(a+c)(b+c)}+\frac{a^2(a-c)(b-c)}{(ab+bc+ca)(a+c)(a+b)}\geq0$
BĐT cuối cùng luôn đúng do $c=min\left \{ a,b,c \right \}$

Đẳng thức xảy ra $\Leftrightarrow a=b=c$




#531791 Topic ôn luyện VMO 2015

Đã gửi bởi chardhdmovies on 04-11-2014 - 19:01 trong Thi HSG Quốc gia và Quốc tế

Bài 7: Cho $x,y,z$ là các số thực không âm đôi một khác nhau. Tìm giá trị nhỏ nhất của biểu thức: 

$(x+y+z)^2.\left [ \frac{1}{(x-y)^2}+\frac{1}{(y-z)^2}+\frac{1}{(z-x)^2} \right ]$

đặt biểu thức là $P$

không mất tính tổng quát giả sử $x>y>z\geq 0$

đặt $x-y=a,y-z=b\Rightarrow z-x=-(a+b)$ với $a,b>0$

ta có $(x+y+z)^2\geq (x+y)^2=\left [ (z+a+b)+(z+b) \right ]^2\geq (a+2b)^2$

ta cần tìm giá trị nhỏ nhất $P=(a+2b)^2\left ( \frac{1}{a^2}+\frac{1}{b^2}+\frac{1}{(a+b)^2} \right )$

đặt $t=\frac{a}{b}>0\Rightarrow P=\left ( 1+\frac{2}{t} \right )^2+(t+2)^2+\left ( 1+\frac{1}{t+1} \right )^2$

tới đây xét hàm ta được $P\geq 9+6\sqrt{3}$ khi $t=\frac{\sqrt{3}}{2}+\frac{\sqrt[4]{3}}{\sqrt{2}}-\frac{1}{2}$

vậy $P_{min}=9+6\sqrt{3}$

dấu bằng xảy ra khi $\boxed{(x,y,z)\sim \left ( \frac{\sqrt{3}}{2}+\frac{\sqrt[4]{3}}{\sqrt{2}}+\frac{1}{2};1;0 \right )}$ và các hoán vị

 

NTP




#532514 Topic ôn luyện VMO 2015

Đã gửi bởi chardhdmovies on 09-11-2014 - 12:51 trong Thi HSG Quốc gia và Quốc tế

Những bài chưa được giải. Lưu ý ai giải được 1 trong số các bài dưới đây mới được tiếp tục đăng đề, còn tự ý đăng đề sẽ bị ẩn + 1 lần nhắc nhở.

Bài 16: Cho $A,B,C$ là 3 góc của một tam giác. CMR:

 

    $cos\frac{A-B}{2}+cos\frac{B-C}{2}+cos\frac{C-A}{2}\geq sin\frac{3A}{2}+sin\frac{3B}{2}+sin\frac{3C}{2}$.

cách 1:

ta có $\left\{\begin{matrix} \sum cos\frac{B-C}{2}=2\sum sin\frac{B}{2}sin\frac{C}{2}+\sum sin\frac{A}{2}\\ \sum sin\frac{3A}{2}=\sum \left ( 3sin\frac{A}{2}-4sin^3\frac{A}{2} \right ) \end{matrix}\right.$ do đó ta cần chứng minh

$2\sum sin\frac{B}{2}sin\frac{C}{2}+4\sum sin^3\frac{A}{2}-\sum sin\frac{A}{2}\geq 0$

mà $\left\{\begin{matrix} 2\sum sin\frac{B}{2}sin\frac{C}{2}=\sum sin\frac{A}{2}\left ( sin\frac{B}{2}+sin\frac{C}{2} \right )\\4\sum sin^3\frac{A}{2}=2\sum \left ( sin^3\frac{B}{2}+sin^3\frac{C}{2} \right ) \\ 2\sum sin\frac{A}{2}=\sum \left ( sin\frac{B}{2}+sin\frac{C}{2} \right ) \end{matrix}\right.$ nên bđt cần chứng minh

$\Leftrightarrow \sum \left [ sin\frac{A}{2}\left ( sin\frac{B}{2}+sin\frac{C}{2} \right )+2\left ( sin^3\frac{B}{2}+sin^3\frac{C}{2} \right )-\left ( sin\frac{B}{2}+sin\frac{C}{2} \right ) \right ]\geq 0$

giờ ta sẽ chứng minh $sin\frac{A}{2}\left ( sin\frac{B}{2}+sin\frac{C}{2} \right )+2\left ( sin^3\frac{B}{2}+sin^3\frac{C}{2} \right )-\left ( sin\frac{B}{2}+sin\frac{C}{2} \right )\geq 0$

$\Leftrightarrow sin\frac{A}{2}+2\left ( sin^2\frac{B}{2}-sin\frac{B}{2}sin\frac{C}{2}+sin^2\frac{C}{2} \right )-1\geq 0$

$\Leftrightarrow sin\frac{A}{2}+(1-cosB)+\left ( cos\frac{B+C}{2}-cos\frac{B-C}{2} \right )+(1-cosC)-1\geq 0$

$\Leftrightarrow 1+2sin\frac{A}{2}\geq cosB+cosC+cos\frac{B-C}{2}$

điều này luôn đúng do $\left\{\begin{matrix} 1\geq cos\frac{B-C}{2}\\2sin\frac{A}{2}\geq cosB+cosC \end{matrix}\right.$

do đó bđt được chứng minh

cách 2:

đặt $x=\frac{A}{2},y=\frac{B}{2},z=\frac{C}{2}\Rightarrow \left\{\begin{matrix} x,y,z\in \left ( 0;\frac{\pi }{2} \right )\\ x+y+z=\frac{\pi }{2} \end{matrix}\right.$

ta có $sin\frac{3A}{2}-cos\frac{B-C}{2}=sin3x-cos(y-z)=sin3x-sin(x+2y)=-2sin(x-y)sin(x-z)$

do đó ta cần chứng minh $\sum sin(x-y)sin(x-z)\geq 0$

không mất tính tổng quát giả sử $0<x\leq y\leq z< \frac{\pi }{2}$

bđt cần chứng minh $\Leftrightarrow sin(x-y)sin(x-z)+sin(z-y)\left [ sin(z-x)-sin(y-x) \right ]\geq 0$

điều này luôn đúng bởi hàm số $y=sin\alpha$ là hàm đồng biến vơi $0<\alpha <\frac{\pi }{2}$

do đó bđt được chứng minh

 

 

NTP




#535448 Topic ôn luyện VMO 2015

Đã gửi bởi chardhdmovies on 30-11-2014 - 06:37 trong Thi HSG Quốc gia và Quốc tế

Bài 54:  Cho $a,b,c>0$. Giải hệ:

$\left\{\begin{matrix}\frac{a}{x}-\frac{b}{z}=c-xz & & \\ \frac{b}{y}-\frac{c}{x}=a-xy & & \\ \frac{c}{z}-\frac{a}{y}=b-yz & & \end{matrix}\right.$

qui đồng rồi cộng các phương trình ta được $axy+byz+czx=x^2y^2+y^2z^2+z^2x^2$        $(1)$

$c.PT(1)+a.PT(2)+b.PT(3)\Rightarrow axy+byz+czx=a^2+b^2+c^2$                             $(2)$

cộng $(1)$ và $(2)$ ta được $(xy-a)^2+(yz-b)^2+(zx-c)^2=0\Rightarrow \left\{\begin{matrix} xy=a\\yz=b \\zx=c \end{matrix}\right.$

phần còn lại ok rồi

Spoiler

 

NTP




#496758 Topic về Bất đẳng thức, cực trị THCS

Đã gửi bởi chardhdmovies on 03-05-2014 - 12:46 trong Bất đẳng thức và cực trị

Góp mấy bài toán:  :wub:  :wub:  :wub: 
Bài 1: Cho các số thực dương $x,y,z$ và thỏa mãn rằng: $x(x+y+z)=3yz$.Chứng minh rằng:

$(x+y)^{3}+(x+z)^{3}+3(x+y)(x+z)(y+z) \leq 5(z+y)^{3}$

 

 

đây là đề thi đại học khối A năm 2009




#522087 Tim min A=$\sum a^2$

Đã gửi bởi chardhdmovies on 31-08-2014 - 12:58 trong Bất đẳng thức và cực trị

xem câu $3$ ở đây

 

                                                                              NTP




#519970 Tim min A=$\sum \frac{x^3-y^3}{(x-y)^3}$

Đã gửi bởi chardhdmovies on 16-08-2014 - 21:43 trong Bất đẳng thức - Cực trị

$A=\sum \frac{x^2+xy+y^2}{(x-y)^2}=\sum \frac{\frac{3}{4}(x+y)^2+\frac{1}{4}(x-y)^2}{(x-y)^2}=\frac{3}{4}\sum \frac{(x+y)^2}{(x-y)^2}+\frac{3}{4}$

phần còn lại em xem VD2 ở đây




#504830 tim min

Đã gửi bởi chardhdmovies on 07-06-2014 - 22:05 trong Bất đẳng thức và cực trị

ta có $\frac{xy}{z}+\frac{yz}{x}\geq 2y$

mấy cái kia tương tự cộng lại là ta được